MATH2111 Higher Several Variable Calculus (1 Viewer)

leehuan

Well-Known Member
Joined
May 31, 2014
Messages
5,805
Gender
Male
HSC
2015
Re: Several Variable Calculus

My lecture slides. I haven't used a maths textbook before at uni. I'm just going to have to take it up with the lecturer
 

leehuan

Well-Known Member
Joined
May 31, 2014
Messages
5,805
Gender
Male
HSC
2015
Re: Several Variable Calculus

Can someone compute the directional derivative at (0,0) of this function?





(Assume u is a unit vector already)
 

seanieg89

Well-Known Member
Joined
Aug 8, 2006
Messages
2,662
Gender
Male
HSC
2007
Re: Several Variable Calculus

The partial derivatives are all zero, but the directional derivatives are as you say for u1 nonzero, and zero if u1=0.

This is one of those funny functions that you don't see the full weirdness of by just looking at 2d slices of its graph. Each partial derivative exists at the origin, but the function f isn't even continuous at the origin!
 

leehuan

Well-Known Member
Joined
May 31, 2014
Messages
5,805
Gender
Male
HSC
2015
They are, but so what? These are not the hypotheses of Clairaut's theorem.
Maybe it was just my unwillingness to compute any second order partial derivatives because the first order partial derivatives were thoroughly untidy.

Is there any easy way of contradicting the criteria of Clairaut's theorem then?
 

glittergal96

Active Member
Joined
Jul 25, 2014
Messages
418
Gender
Female
HSC
2014
Sorry about alt, am on phone. Showing that typical Clairaut hypotheses don't hold doesn't prove that mixed partials differ because its not iff. Just compute mixed partials at 0, it really shouldn't take long.

Sent from my GT-I9506 using Tapatalk
 

leehuan

Well-Known Member
Joined
May 31, 2014
Messages
5,805
Gender
Male
HSC
2015
Oh, that was done in a previous part. I understand your point there...



...albeit this was the question. It's the last part that's of interest.
 

InteGrand

Well-Known Member
Joined
Dec 11, 2014
Messages
6,109
Gender
Male
HSC
N/A
Oh, that was done in a previous part. I understand your point there...



...albeit this was the question. It's the last part that's of interest.
It didn't work because the hypotheses of Clairaut's Theorem were not satisfied by that ƒ (I assume you know what typical hypotheses are for it).
 
Last edited:

leehuan

Well-Known Member
Joined
May 31, 2014
Messages
5,805
Gender
Male
HSC
2015
It didn't work because the hypotheses of Clariaut's Theorem were not satisfied by that ƒ (I assume you know what typical hypotheses are for it).
Goes back up to the above query I suppose. Is the only way of really proving it formally just to brute those second order derivatives?
 

InteGrand

Well-Known Member
Joined
Dec 11, 2014
Messages
6,109
Gender
Male
HSC
N/A
Goes back up to the above query I suppose. Is the only way of really proving it formally just to brute those second order derivatives?
You could probably also do it by computing the second partial derivatives at (x,y) ≠ (0, 0) and investigating their limit as (x, y) -> (0, 0), but there's no need to do that since you've already computed the mixed second-partials at the origin in an earlier part of the question (part b)).
 

leehuan

Well-Known Member
Joined
May 31, 2014
Messages
5,805
Gender
Male
HSC
2015
I suppose what's really bugging me is the fact that d) is actually asking to explain why b) is true. Looks like more of just a grind rather than testing some nice stuff but I suppose I'll just have to deal with it.
 

seanieg89

Well-Known Member
Joined
Aug 8, 2006
Messages
2,662
Gender
Male
HSC
2007
It really doesn't take that long, especially since to prove discontinuity you can just show that approaching the origin via different axes gives a different result, so most terms in the mixed partials drop out from x or y being zero.

I would definitely go via this route rather than by restating the fact that the mixed partials do not match and using the contrapositive of Clairaut based on the wording of the question.

"Because the conclusion of Clairaut's theorem is not true" is NOT what I would consider a good answer to "Why does Clairaut's theorem not work here?".
 

leehuan

Well-Known Member
Joined
May 31, 2014
Messages
5,805
Gender
Male
HSC
2015




c) Give a simple formula for f(x)

 
Last edited:

Users Who Are Viewing This Thread (Users: 0, Guests: 1)

Top